Question:
Dérivation des équations de Maxwell à partir du tenseur de champ lagrangien
amc
2011-01-16 14:18:20 UTC
view on stackexchange narkive permalink

J'ai commencé à lire Peskin et Schroeder pendant mon propre temps, et je ne sais pas trop comment obtenir les équations de Maxwell à partir de la densité lagrangienne (sans source) $ L = - \ frac {1} {4} F _ {\ mu \ nu} F ^ {\ mu \ nu} $ (où $ F ^ {\ mu \ nu} = \ partial ^ \ mu A ^ \ nu - \ partial ^ \ nu A ^ \ mu $ est le tenseur de champ).

En substituant à la définition du tenseur de champ, on obtient $ L = - \ frac {1} {2} [(\ partial_ \ mu A_ \ nu) (\ partial ^ \ mu A ^ \ nu) - (\ partial_ \ mu A_ \ nu) (\ partial ^ \ nu A ^ \ mu)] $. Je sais que je devrais utiliser $ A ^ \ mu $ comme variable dynamique dans les équations d'Euler-Lagrange, qui deviennent $ \ frac {\ partial L} {\ partial A_ \ mu} - \ partial_ \ mu \ frac {\ partial L} {\ partial (\ partial_ \ mu A_ \ nu)} = - \ partial_ \ mu \ frac {\ partial L} {\ partial (\ partial_ \ mu A_ \ nu)} $, mais je ne sais pas comment pour continuer à partir d'ici.

Je sais que je devrais me retrouver avec $ \ partial_ \ mu F ^ {\ mu \ nu} = 0 $, mais je ne vois pas très bien pourquoi. Puisque $ \ mu $ et $ \ nu $ sont des indices fictifs, je devrais pouvoir les changer: comment les indices dans le lagrangien se rapportent-ils aux indices des dérivées dans les équations d'Euler-Lagrange?

Voir dans le livre de Sean carroll.Plein de dérivation là-bas
Pourquoi n'est-il pas suffisant de brancher les $ F ^ {\ mu \ nu} = \ partial {[\ mu} A {\ nu]} $ dans les équations de Maxwell et de montrer qu'elles tiennent?
Cinq réponses:
Pablo
2011-09-19 18:36:46 UTC
view on stackexchange narkive permalink

Nous faisons varier l'action $$ \ delta \ int {L \; \ mathrm {d} t} = \ delta \ int {\ int {\ Lambda \ left ({A_ \ nu, \ partial _ \ mu A_ \ nu} \ right) \ mathrm {d} ^ 3 x \; \ mathrm {d} t = 0}} $$$ {\ Lambda \ left ({A_ \ nu, \ partial _ \ mu A_ \ nu} \ right )} $ est la densité de lagrangien du système.

Donc, $$ \ int {\ int {\ left ({\ frac {{\ partial \ Lambda}} {{\ partial A_ \ nu }} \ delta A_ \ nu + \ frac {{\ partial \ Lambda}} {{\ partial \ left ({\ partial _ \ mu A_ \ nu} \ right)}} \ delta \ left ({\ partial _ \ mu A_ \ nu} \ right)} \ right) \ mathrm {d} ^ 3 x \; \ mathrm {d} t = 0}} $$ En intégrant par parties on obtient: $$ \ int {\ int {\ gauche ({\ frac {{\ partial \ Lambda}} {{\ partial A_ \ nu}} - \ partial _ \ mu \ frac {{\ partial \ Lambda}}} {{\ partial \ left ({\ partial _ \ mu A_ \ nu} \ right)}}} \ right) \ delta A_ \ nu \ mathrm {d} ^ 3 x \; \ mathrm {d} t = 0}} \ implique \ frac {{\ partial \ Lambda} } {{\ partial A_ \ nu}} - \ partial _ \ mu \ frac {{\ partial \ Lambda}} {{\ partial \ left ({\ partial _ \ mu A_ \ nu} \ right)}} = 0 $$ Nous devons déterminer la densité du lagrangien. Un des termes traite de l'interaction des charges avec le champ électromagnétique, $ J ^ \ mu A_ \ mu $. L'autre terme est la densité d'énergie du champ électromagnétique: ce terme est la différence du champ magnétique et du champ électrique. Nous avons donc: $$ \ Lambda = J ^ \ mu A_ \ mu + \ frac {1} {{4 \ mu _0}} F ^ {\ mu \ nu} F _ {\ mu \ nu} $$ Nous avons: $$ \ frac {{\ partial \ Lambda}} {{\ partial A_ \ nu}} = J ^ \ nu $$ donc: \ begin {align} \ partial _ \ mu \ frac {{\ partial \ Lambda}} {{\ partial \ left ({\ partial _ \ mu A_ \ nu} \ right)}} & = \ frac {1} { {4 \ mu _0}} \ partial _ \ mu \ left ({\ frac {\ partial} {{\ partial \ left ({\ partial _ \ mu A_ \ nu} \ right)}} F ^ {\ kappa \ lambda} F _ {\ kappa \ lambda}} \ right) \\ & = \ frac {1} {{4 \ mu _0}} \ partial _ \ mu \ left ({\ frac {\ partial} {{\ partial \ left ({\ partial _ \ mu A_ \ nu} \ right)}} \ left ({\ left ({\ partial ^ \ kappa A ^ \ lambda - \ partial ^ \ lambda A ^ \ kappa} \ right) \ left ({\ partial _ \ kappa A_ \ lambda - \ partial _ \ lambda A_ \ kappa} \ right)} \ right)} \ right) \\ & = \ frac {1} {{4 \ mu _0}} \ partial _ \ mu \ left ({\ frac {\ partial} {{\ partial \ left ({\ partial _ \ mu A_ \ nu} \ right)}} \ left ({\ partial ^ \ kappa A ^ \ lambda \ partial _ \ kappa A_ \ lambda - \ partial ^ \ kappa A ^ \ lambda \ partial _ \ lambda A_ \ kappa - \ partial ^ \ lambda A ^ \ kappa \ partial _ \ kappa A_ \ lambda + \ partial ^ \ lambda A ^ \ kappa \ partial _ \ lambda A_ \ kappa} \ right)} \ right) \ end {align} Le troisième et le quatrième sont les mêmes des premier et deuxième termes. Vous pouvez faire $ k \ leftrightarrow \ lambda $: \ begin {align} \ partial _ \ mu \ frac {{\ partial \ Lambda}} {{\ partial \ left ({\ partial _ \ mu A_ \ nu} \ right )}} & = \ frac {1} {{2 \ mu _0}} \ partial _ \ mu \ left ({\ frac {\ partial} {{\ partial \ left ({\ partial _ \ mu A_ \ nu} \ right)}} \ left ({\ partial ^ \ kappa A ^ \ lambda \ partial _ \ kappa A_ \ lambda - \ partial ^ \ kappa A ^ \ lambda \ partial _ \ lambda A_ \ kappa} \ right)} \ right) \;. \ end {align} Mais \ begin {align} \ frac {\ partial} {{\ partial \ left ({\ partial _ \ mu A_ \ nu} \ right)}} \ left ({\ partial ^ \ kappa A ^ \ lambda \ partial _ \ kappa A_ \ lambda} \ right) & = \ partial ^ \ kappa A ^ \ lambda \ frac {\ partial} {{\ partial \ left ({\ partial _ \ mu A_ \ nu} \ right)}} \ left ( {\ partial _ \ kappa A_ \ lambda} \ right) + \ partial _ \ kappa A_ \ lambda \ frac {\ partial} {{\ partial \ left ({\ partial _ \ mu A_ \ nu} \ right)}} \ left ({\ partial ^ \ kappa A ^ \ lambda} \ right) \\ & = \ partial ^ \ kappa A ^ \ lambda \ delta _ \ kappa ^ \ mu \ delta _ \ lambda ^ \ nu + g ^ { \ kappa \ alpha} g ^ {\ lambda \ beta} \ partial _ \ kappa A_ \ lambda \ frac {\ partial} {{\ partial \ left ({\ partial _ \ mu A_ \ nu} \ right)}} \ left ({\ partial _ \ alpha A_ \ beta} \ right) \\ & = 2 \ partial ^ \ mu A ^ \ nu \;. \ end {align}

Nous avons:

\ begin {align} \ frac {\ partial} {{\ partial \ left ({\ partial _ \ mu A_ \ nu} \ right)}} \ left ({\ partial ^ \ kappa A ^ \ lambda \ partial _ \ lambda A_ \ kappa} \ right) & = 2 \ partial ^ \ nu A ^ \ mu \ ;. \ end {align}

Donc,

\ begin {align} \ partial _ \ mu \ left ({\ frac {{\ partial \ Lambda}} {{\ partial \ gauche ({\ partial _ \ mu A_ \ nu} \ right)}}} \ right) & = \ frac {1} {{\ mu _0}} \ partial _ \ mu \ left ({\ partial ^ \ mu A ^ \ nu - \ partial ^ \ nu A ^ \ mu} \ right) \\ & = \ frac {1} {{\ mu _0}} \ partial _ \ mu F ^ {\ mu \ nu} \;. \ end {align} Les équations lagrangiennes fournissent les équations de maxwell non homogènes:

$$ \ partial _ \ mu F ^ {\ mu \ nu} = \ mu _0 J ^ \ nu \ ;. $$

Pour info, cette réponse (v4) utilise implicitement la convention de signe $ (+, -, -, -) $.
Marek
2011-01-16 14:46:19 UTC
view on stackexchange narkive permalink

Eh bien, vous y êtes presque. Utilisez le fait que $$ {\ partial (\ partial _ {\ mu} A _ {\ nu}) \ over \ partial (\ partial _ {\ rho} A _ {\ sigma})} = \ delta _ {\ mu} ^ {\ rho} \ delta _ {\ nu} ^ {\ sigma} $$ qui est valide car $ \ partial _ {\ mu} A _ {\ nu} $ sont $ d ^ 2 $ composants indépendants.

Comment le prouver réellement?
Luboš Motl
2011-01-16 15:09:38 UTC
view on stackexchange narkive permalink

Cher amc, tout d'abord, écrivez votre densité lagrangienne comme $$ L = - \ frac {1} {4} F _ {\ mu \ nu} F ^ {\ mu \ nu} = - \ frac {1} {2 } (\ partial_ \ mu A_ \ nu) F ^ {\ mu \ nu} $$ Est-ce que ça va jusqu'à présent? Le $ F _ {\ mu \ nu} $ contient deux termes qui le rendent antisymétrique dans les deux indices. Cependant, il est multiplié par un autre $ F ^ {\ mu \ nu} $ qui est déjà antisymétrique, donc je n'ai pas besoin de l'antisymétriser à nouveau. Au lieu de cela, les deux termes me donnent la même chose, donc le coefficient $ -1 / 4 $ change simplement en $ -1 / 2 $.

Maintenant, les équations de champ vous obligent à calculer les dérivées du lagrangien par rapport à $ A_ \ mu $ et ses dérivés. Tout d'abord, la dérivée du lagrangien $ L $ par rapport aux composants $ A_ \ mu $ eux-mêmes disparaît car le lagrangien ne dépend que des dérivées partielles de $ A_ \ mu $. Est-ce clair jusqu'ici?

Les équations du mouvement seront donc $$ 0 = - \ partial_ \ mu [\ partial L / \ partial (\ partial_ \ mu A_ \ nu)] = \ dots $$ Oups, vous êtes déjà arrivé à ce point. Mais maintenant, regardez ma forme du lagrangien ci-dessus. La dérivée du lagrangien par rapport à $ \ partial_ \ mu A_ \ nu $ est simplement $$ - \ frac {1} {2} F ^ {\ mu \ nu} $$ car $ \ partial_ \ mu A_ \ nu $ apparaît simplement comme un facteur donc les équations du mouvement seront simplement $$ 0 = + \ frac {1} {2} \ partial_ \ mu F ^ {\ mu \ nu} $$ Cependant, j'ai délibérément commis une erreur. Je n'ai différencié le lagrangien que par rapport à $ \ partial_ \ mu A_ \ nu $ inclus dans le premier facteur de $ F _ {\ mu \ nu} $, avec les indices inférieurs. Cependant, les composants $ \ partial_ \ mu A_ \ nu $ apparaissent également dans $ F ^ {\ mu \ nu} $, le deuxième facteur du lagrangien, celui avec les indices supérieurs. Si vous ajoutez les termes correspondants de la règle de Leibniz, le résultat est simplement que la contribution entière doublera. Ainsi, la bonne équation de mouvement, y compris le coefficient naturel, sera $$ 0 = \ partial_ \ mu F ^ {\ mu \ nu} $$ La normalisation globale est importante car cette équation peut obtenir des termes supplémentaires, comme le courant, dont le coefficient est évident, et vous ne voulez pas obtenir une erreur relative de deux entre la dérivée de $ F $ et le $ j $ courant.

CheersLubos

Hé, je sais que c'est 5 ans de retard mais peut-être que vous verrez ceci: Pourquoi $ \ frac {\ partial} {\ partial (\ partial _ {\ mu} \ phi)} (\ partial _ {\ mu} A _ {\ nu} F ^ {\ mu \ nu}) = F ^ {\ mu \ nu} $.Le tenseur ne dépend-il pas aussi des dérivées partielles?Ne devons-nous pas utiliser la règle du produit alors?
Salut @user17574 - "quel" tenseur ne dépend-il pas des dérivées partielles?Sûrement le tenseur énergie-contrainte le fait, tout comme le lagrangien.C'est pourquoi la dérivée de celui-ci par rapport aux dérivées partielles est non nulle.Le dérivé est calculé dans la réponse.La règle du produit fonctionne en effet et c'est pourquoi on annule le facteur de 1/2 $.Avez-vous essayé de lire la réponse?
Je sais que c'est une vieille question, mais j'ai un doute.Après avoir appliqué la règle de Leibniz, nous obtenons: $$ \ frac {\ partial \ mathcal {L}} {\ partial (\ partial_ \ mu A_ \ nu)} = - \ frac {1} {2} \ left (\ partial_ \ mu A_ \ nu \ frac {\partial F ^ {\ mu \ nu}} {\ partial (\ partial_ \ mu A_ \ nu)} + \ frac {\ partial_ \ mu A_ \ nu} {\ partial (\ partial_ \ mu A_ \ nu)} F ^{\ mu \ nu} \ right) $$ Comment suis-je censé différencier le $ F ^ {\ mu \ nu} $ sur le premier terme de l'équation?Je ne comprends pas comment cela me donnera $ F ^ {\ mu \ nu} $.
Vous devez utiliser une nouvelle paire d'indices au lieu de copier mu-nu trois fois.Alors $ \ partial F ^ {\ alpha \ beta} / \ partial (\ partial_ \ gamma A_ \ delta) = g ^ {\ alpha \ gamma} g ^ {\ beta \ delta} - \ alpha \ leftrightarrow \ beta $ simplementcar $ F $ est juste la différence entre deux termes similaires (antisymétrisation) et chaque terme a la dérivée qui est fondamentalement un delta de Kronecker - mais ici avec les indices élevés (pour devenir la métrique de l'indice supérieur) en raison de l'emplacement des indicesdans l'expression originale.
Frobenius
2017-06-07 12:44:04 UTC
view on stackexchange narkive permalink

Bien que tard dans la fête, je poste une réponse à un niveau élémentaire. Peut-être que cela prouve la puissance du calcul tensoriel utilisé dans toutes les bonnes réponses précédentes.

Abstract

Dans cette réponse, nous essaierons de dériver des équations de Maxwell dans un espace vide \ begin {align} \ boldsymbol {\ nabla} \ boldsymbol {\ times} \ mathbf {E} & = - \ frac {\ partial \ mathbf {B}} {\ partial t} \ tag {001a} \\ \ boldsymbol {\ nabla} \ boldsymbol {\ times} \ mathbf {B} & = \ mu_ {0} \ mathbf {j} + \ frac {1} {c ^ {2}} \ frac {\ partial \ mathbf { E}} {\ t partiel} \ tag {001b} \\ \ nabla \ boldsymbol {\ cdot} \ mathbf {E} & = \ frac {\ rho} {\ epsilon_ {0}} \ tag {001c} \\ \ nabla \ boldsymbol {\ cdot} \ mathbf {B} & = 0 \ tag {001d} \ end {align} à partir des équations d'Euler-Lagrange \ begin {équation} \ boxed {\: \ dfrac {\ partial} {\ partial t} \ left (\ dfrac {\ partial \ mathcal {L}} {\ partial \ dot {\ eta} _ {\ jmath}} \ right) + \ nabla \ boldsymbol {\ cdot} \ left [\ dfrac {\ partial \ mathcal {L}} {\ partial \ left (\ boldsymbol {\ nabla} \ eta _ {\ jmath} \ right)} \ right] - \ frac {\ partial \ mathcal { L}} {\ partial \ eta _ {\ jmath}} = 0, \ quad \ left (\ jmath = 1,2,3,4 \ right) \:} \ tag {002} \ end {équation} où \ begin {équation} \ mathcal {L} = \ mathcal {L} \ left (\ eta _ {\ jmath}, \ dot {\ eta} _ {\ jmath}, \ boldsymbol {\ nabla} \ eta _ {\ jmath} \ right) \ qquad \ gauche (\ jmath = 1,2,3,4 \ droite) \ tag {003} \ end {équation} est la densité lagrangienne de la question (sauf un facteur constant) \ begin {équation} \ boxed {\: \ mathcal {L} = \ dfrac {\ Vert \ mathbf {E} \ Vert ^ {2} -c ^ {2} \ Vert \ mathbf {B} \ Vert ^ {2}} {2} + \ dfrac {1 } {\ epsilon_ {0}} \ left (- \ rho \ phi + \ mathbf {j} \ boldsymbol {\ cdot} \ mathbf {A} \ right) \:} \ tag {004} \ end {équation} et $ \: \ eta _ {\ jmath} \ left (x_ {1}, x_ {2}, x_ {3}, t \ right), \: \: \ jmath = 1,2,3,4 \: $ les composantes $ \: A_ {1}, \: A_ {2}, \: A_ {3}, \ phi \: $ du 4-vecteur potentiel EM respectivement. Dans un sens, cette dérivation est construite sur l'inverse (: celle de trouver une densité lagrangienne correcte à partir des équations de Maxwell) en reculant, voir ma réponse ici: Deriving Lagrangian densité for electromagnetic field

1. Section principale

D'abord, nous exprimons $ \: \ mathbf {E}, \ mathbf {B} \: $ de (004) en termes de composantes potentielles à 4 vecteurs $ \: A_ {1}, \: A_ {2} , \: A_ {3}, \ phi \: $ \ begin {align} \ mathbf {B} & = \ boldsymbol {\ nabla} \ boldsymbol {\ times} \ mathbf {A} \ tag {005a} \\ \ mathbf {E} & = - \ boldsymbol {\ nabla} \ phi - \ dfrac {\ partial \ mathbf {A}} {\ partial t} = - \ boldsymbol {\ nabla} \ phi - \ mathbf {\ dot { UNE}} \ tag {005b} \ end {align} A partir de (005) les équations de Maxwell (001a) et (001d) sont valides automatiquement. Les quatre (4) équations scalaires de Maxwell (001b) et (001c) doivent donc être dérivées des quatre (4) équations scalaires d'Euler-Lagrange (002). De plus, il est raisonnable de supposer que l'équation vectorielle (001b) doit être dérivée de (002) par rapport aux composantes du potentiel vectoriel $ \: \ mathbf {A} = \ left (A_ {1}, \: A_ { 2}, \: A_ {3} \ right) \: $, tandis que l'équation scalaire (001c) doit être dérivée de (002) par rapport au potentiel scalaire $ \: \ phi \: $.

À partir des équations (005), nous exprimons la densité lagrangienne (004) en termes de composantes potentielles à 4 vecteurs $ \: A_ {1}, \: A_ {2}, \: A_ {3}, \ phi \ : $: \ begin {align} \ left \ Vert \ mathbf {E} \ right \ Vert ^ {2} & = \ left \ Vert - \ boldsymbol {\ nabla} \ phi - \ dfrac {\ partial \ mathbf {A}} {\ partial t} \ droite \ Vert ^ {2} = \ gauche \ Vert \ mathbf {\ dot {A}} \ droite \ Vert ^ {2} + \ Vert \ boldsymbol {\ nabla} \ phi \ Vert ^ {2} +2 \ gauche (\ boldsymbol {\ nabla} \ phi \ boldsymbol {\ cdot} \ mathbf {\ dot {A}} \ right) \ tag {006a} \\ & \pas de numéro\\ \ left \ Vert \ mathbf {B} \ right \ Vert ^ {2} & = \ left \ Vert \ boldsymbol {\ nabla} \ boldsymbol {\ times} \ mathbf {A} \ right \ Vert ^ {2} \ equiv \ sum ^ {k = 3} _ {k = 1} \ left [\ Vert \ boldsymbol {\ nabla} \ mathrm {A} _ {k} \ Vert ^ {2} - \ dfrac {\ partial \ mathbf {A }} {\ partial x_ {k}} \ boldsymbol {\ cdot} \ boldsymbol {\ nabla} \ mathrm {A} _ {k} \ right] \ tag {006b} \ end {align} La deuxième équation de (006b), c'est-à-dire l'identité \ begin {équation} \ left \ Vert \ boldsymbol {\ nabla} \ boldsymbol {\ times} \ mathbf {A} \ right \ Vert ^ {2} \ equiv \ sum ^ {k = 3} _ {k = 1} \ left [\ Vert \ boldsymbol {\ nabla} \ mathrm {A} _ {k} \ Vert ^ {2} - \ dfrac {\ partial \ mathbf {A}} {\ partial x_ {k}} \ boldsymbol {\ cdot} \ boldsymbol { \ nabla} \ mathrm {A} _ {k} \ right] \ tag {Id-01} \ end {équation} est prouvé dans 2. Section des identités. En insérant des expressions (006) dans (004), la densité lagrangienne est \ begin {équation} \ mathcal {L} = \ underbrace {\ tfrac {1} {2} \ left \ Vert \ mathbf {\ dot {A}} \ right \ Vert ^ {2} + \ tfrac {1} {2} \ Vert \ boldsymbol {\ nabla} \ phi \ Vert ^ {2} + \ boldsymbol {\ nabla} \ phi \ boldsymbol {\ cdot} \ mathbf {\ dot {A}}} _ {\ tfrac {1} {2} \ left \ Vert - \ boldsymbol {\ nabla} \ phi - \ frac {\ partial \ mathbf {A}} {\ partial t} \ right \ Vert ^ {2}} - \ tfrac {1} {2} c ^ {2 } \ underbrace {\ sum ^ {k = 3} _ {k = 1} \ left [\ Vert \ boldsymbol {\ nabla} \ mathrm {A} _ {k} \ Vert ^ {2} - \ frac {\ partial \ mathbf {A}} {\ partial x_ {k}} \ boldsymbol {\ cdot} \ boldsymbol {\ nabla} \ mathrm {A} _ {k} \ right]} _ {\ gauche \ Vert \ boldsymbol {\ nabla } \ boldsymbol {\ times} \ mathbf {A} \ right \ Vert ^ {2}} + \ frac {1} {\ epsilon_ {0}} \ left (- \ rho \ phi + \ mathbf {j} \ boldsymbol {\ cdot} \ mathbf {A} \ droite) \ tag {007} \ end {équation}

Nous réorganisons les éléments de (007) comme suit:

\ begin {align} \ mathcal {L} & = \ overbrace {\ tfrac {1} {2} \ Vert \ boldsymbol {\ nabla} \ phi \ Vert ^ {2} - \ frac {\ rho \ phi} {\ epsilon_ {0}} + \ boldsymbol {\ nabla} \ phi \ boldsymbol {\ cdot} \ mathbf {\ dot {A}}} ^ {\ mathcal {L} _ {\ phi} = \ text {par rapport à} \ phi} + \ tfrac {1} {2} \ left \ Vert \ mathbf {\ dot {A}} \ right \ Vert ^ {2} + \ tfrac {1} {2} c ^ {2} \ sum ^ {k = 3} _ {k = 1} \ left [\ frac {\ partial \ mathbf {A}} {\ partial x_ {k}} \ boldsymbol {\ cdot} \ boldsymbol {\ nabla} \ mathrm {A} _ {k} - \ Vert \ boldsymbol {\ nabla} \ mathrm {A} _ {k} \ Vert ^ {2} \ right] + \ frac {\ mathbf {j} \ boldsymbol {\ cdot} \ mathbf {A}} {\ epsilon_ {0}} \ tag {008a} \\ \ mathcal {L} & = \ tfrac {1} {2} \ Vert \ boldsymbol {\ nabla} \ phi \ Vert ^ {2} - \ frac {\ rho \ phi} {\ epsilon_ {0}} + \ underbrace {\ boldsymbol {\ nabla} \ phi \ boldsymbol {\ cdot} \ mathbf {\ dot {A}} + \ tfrac {1} {2} \ left \ Vert \ mathbf {\ dot {A}} \ right \ Vert ^ {2} + \ tfrac {1} {2} c ^ {2} \ sum ^ {k = 3} _ {k = 1} \ left [\ frac {\ partial \ mathbf {A}} {\ partial x_ {k}} \ boldsymbol {\ cdot} \ boldsymbol {\ nabla} \ mathrm {A} _ {k} - \ Vert \ boldsymbol {\ nabla} \ mathrm {A} _ {k} \ Vert ^ {2} \ droite] + \ frac {\ mathbf {j} \ boldsymbol {\ cdot} \ mathbf {A}} {\ epsilon_ {0}}} _ {\ mathcal {L} _ {\ mathbf {A}} = \ text { par rapport à} \ mathbf {A}} \ tag {008b} \ end {align}

La partie $ \: \ mathcal {L} _ {\ phi} \: $ de la densité contient tous les termes $ \: \ phi $ et participera raisonnablement seule à la dérivation de l'équation de Maxwell (001c) à partir de l'équation d'Euler-Lagrange (002) par rapport à $ \: \ eta_ {4} = \ phi \: $. La partie $ \: \ mathcal {L} _ {\ mathbf {A}} \: $ de la densité contient tous les termes $ \: \ mathbf {A} $ - et participera raisonnablement seule à la dérivation de l'équation de Maxwell ( 001b) des équations d'Euler-Lagrange (002) par rapport à $ \: \ eta_ {1}, \ eta_ {2}, \ eta_ {3} = A_ {1}, A_ {1}, A_ {3} \ : $. Notez le terme courant $ \: \ boldsymbol {\ nabla} \ phi \ boldsymbol {\ cdot} \ mathbf {\ dot {A}} \: $ des parties $ \: \ mathcal {L} _ {\ phi}, \ mathcal {L} _ {\ mathbf {A}} \: $.

L'équation d'Euler-Lagrange par rapport à $ \: \ eta_ {4} = \ phi \: $ est: \ begin {équation} \ dfrac {\ partial} {\ partial t} \ overbrace {\ left (\ dfrac {\ partial \ mathcal {L}} {\ partial \ dot {\ phi}} \ right)} ^ {0} + \ nabla \ boldsymbol {\ cdot} \ overbrace {\ left [\ dfrac {\ partial \ mathcal {L}} {\ partial \ left (\ boldsymbol {\ nabla} \ phi \ right)} \ right]} ^ {\ boldsymbol {\ nabla} \ phi + \ mathbf {\ dot {A}}} - \ overbrace {\ frac {\ partial \ mathcal {L}} {\ partial \ phi}} ^ {- \ frac {\ rho} {\ epsilon_ {0 }}} = 0 \ tag {009} \ end {équation} ou \ begin {équation} \ nabla \ boldsymbol {\ cdot} \ underbrace {\ left (- \ boldsymbol {\ nabla} \ phi - \ frac {\ partial \ mathbf {A}} {\ partial t} \ right)} _ {\ mathbf {E }} = \ frac {\ rho} {\ epsilon_ {0}} \ tag {010} \ end {équation} c'est l'équation de Maxwell (001c) \ begin {équation} \ nabla \ boldsymbol {\ cdot} \ mathbf {E} = \ frac {\ rho} {\ epsilon_ {0}} \ tag {001c} \ end {équation}

Pour dériver l'équation de Maxwell (001b), nous l'exprimons à l'aide des équations (005) en termes de composantes potentielles à 4 vecteurs $ \: A_ {1}, \: A_ {2}, \: A_ {3}, \ phi \: $: \ begin {équation} \ boldsymbol {\ nabla} \ boldsymbol {\ times} \ left (\ boldsymbol {\ nabla} \ boldsymbol {\ times} \ mathbf {A} \ right) = \ mu_ {0} \ mathbf {j} + \ frac { 1} {c ^ {2}} \ frac {\ partial} {\ partial t} \ left (- \ boldsymbol {\ nabla} \ phi - \ frac {\ partial \ mathbf {A}} {\ partial t} \ droite) \ tag {011} \ end {équation} Utiliser l'identité \ begin {équation} \ boldsymbol {\ nabla} \ boldsymbol {\ times} \ left (\ boldsymbol {\ nabla} \ boldsymbol {\ times} \ mathbf {A} \ right) = \ boldsymbol {\ nabla} \ left (\ nabla \ boldsymbol { \ cdot} \ mathbf {A} \ right) - \ nabla ^ {2} \ mathbf {A} \ tag {012} \ end {équation} l'équation (011) donne \ begin {équation} \ frac {1} {c ^ {2}} \ frac {\ partial ^ {2} \ mathbf {A}} {\ partial t ^ {2}} - \ nabla ^ {2} \ mathbf {A} + \ boldsymbol {\ nabla} \ left (\ nabla \ boldsymbol {\ cdot} \ mathbf {A} + \ frac {1} {c ^ {2}} \ frac {\ partial \ phi} {\ partial t} \ right) = \ mu_ {0} \ mathbf {j} \ tag {013} \ end {équation} Le composant $ \: k $ de l'équation (013) est exprimé correctement pour ressembler à une équation d'Euler-Lagrange comme suit: \ begin {équation} \ dfrac {\ partial} {\ partial t} \ left (\ frac {\ partial \ mathrm {A} _ {k}} {\ partial t} + \ frac {\ partial \ phi} {\ partial x_ {k} } \ right) + \ nabla \ boldsymbol {\ cdot} \ left [c ^ {2} \ left (\ frac {\ partial \ mathbf {A}} {\ partial x_ {k}} - \ boldsymbol {\ nabla} \ mathrm {A} _ {k} \ right) \ right] - \ frac {\ mathrm {j} _ {k}} {\ epsilon_ {0}} = 0 \ tag {014} \ end {équation} Il suffit d'atteindre au-dessus de l'eq. (014) de l'équation d'Euler-Lagrange (002) avec respect $ \: \ eta_ {k} = A_ {k}, \: \: k = 1,2,3 \: $:

\ begin {équation} \ dfrac {\ partial} {\ partial t} \ left (\ dfrac {\ partial \ mathcal {L}} {\ partial \ dot {A} _ {k}} \ right) + \ nabla \ boldsymbol {\ cdot} \ left [\ dfrac {\ partial \ mathcal {L}} {\ partial \ left (\ boldsymbol {\ nabla} A_ {k} \ right)} \ right] - \ frac {\ partial \ mathcal {L}} { \ partial A_ {k}} = 0 \ tag {015} \ end {équation}

Maintenant \ begin {équation} \ dfrac {\ partial \ mathcal {L}} {\ partial \ dot {A} _ {k}} = \ dfrac {\ partial} {\ partial \ dot {A} _ {k}} \ left (\ boldsymbol { \ nabla} \ phi \ boldsymbol {\ cdot} \ mathbf {\ dot {A}} + \ tfrac {1} {2} \ left \ Vert \ mathbf {\ dot {A}} \ right \ Vert ^ {2} \ right) = \ frac {\ partial \ phi} {\ partial x_ {k}} + \ frac {\ partial \ mathrm {A} _ {k}} {\ partial t} \ tag {016a} \ end {équation}

\ begin {équation} \ frac {\ partial \ mathcal {L}} {\ partial A_ {k}} = \ frac {\ partial} {\ partial A_ {k}} \ left (\ frac {\ mathbf {j} \ boldsymbol {\ cdot } \ mathbf {A}} {\ epsilon_ {0}} \ right) = \ frac {\ mathrm {j} _ {k}} {\ epsilon_ {0}} \ tag {016b} \ end {équation} et \ begin {équation} \ dfrac {\ partial \ mathcal {L}} {\ partial \ left (\ boldsymbol {\ nabla} A_ {k} \ right)} = \ dfrac {\ partial} {\ partial \ left (\ boldsymbol {\ nabla} A_ {k} \ right)} \ left (\ tfrac {1} {2} c ^ {2} \ sum ^ {k = 3} _ {k = 1} \ left [\ frac {\ partial \ mathbf {A }} {\ partial x_ {k}} \ boldsymbol {\ cdot} \ boldsymbol {\ nabla} \ mathrm {A} _ {k} - \ Vert \ boldsymbol {\ nabla} \ mathrm {A} _ {k} \ Vert ^ {2} \ right] \ right) = c ^ {2} \ left (\ frac {\ partial \ mathbf {A}} {\ partial x_ {k}} - \ boldsymbol {\ nabla} \ mathrm {A } _ {k} \ droite) \ tag {016c} \ end {équation} La dernière équation de (016c) est valide en raison de l'identité (Id-02) prouvée dans 2. Section des identités: \ begin {équation} \ dfrac {\ partial \ left (\ left | \! \ left | \ boldsymbol {\ nabla} \ boldsymbol {\ times} \ mathbf {A} \ right | \! \ right | ^ {2} \ right)} { \ partial \ left (\ boldsymbol {\ nabla} \ mathrm {A} _ {k} \ right)} = \ dfrac {\ partial} {\ partial \ left (\ boldsymbol {\ nabla} \ mathrm {A} _ { k} \ right)} \ left (\ sum ^ {k = 3} _ {k = 1} \ left [\ frac {\ partial \ mathbf {A}} {\ partial x_ {k}} \ boldsymbol {\ cdot } \ boldsymbol {\ nabla} \ mathrm {A} _ {k} - \ Vert \ boldsymbol {\ nabla} \ mathrm {A} _ {k} \ Vert ^ {2} \ right] \ right) = 2 \ left (\ boldsymbol {\ nabla} \ mathrm {A} _ {k} - \ frac {\ partial \ mathbf {A}} {\ partial x_ {k}} \ right) \ tag {Id-02} \ end {équation} En utilisant les expressions des équations (016), l'équation d'Euler-Lagrange (015) donne (014) et donc l'équation de Maxwell (001b).

2. Section des identités

Si $ \: \ mathbf {A} = \ left (\ mathrm {A} _ {1}, \ mathrm {A} _ {2}, \ mathrm {A} _ {3} \ right) \: $ est un fonction vectorielle des coordonnées cartésiennes $ \: \ left (x_ {1}, x_ {2}, x_ {3} \ right) \: $ puis \ begin {équation} \ left \ Vert \ boldsymbol {\ nabla} \ boldsymbol {\ times} \ mathbf {A} \ right \ Vert ^ {2} \ equiv \ sum ^ {k = 3} _ {k = 1} \ left [\ Vert \ boldsymbol {\ nabla} \ mathrm {A} _ {k} \ Vert ^ {2} - \ dfrac {\ partial \ mathbf {A}} {\ partial x_ {k}} \ boldsymbol {\ cdot} \ boldsymbol { \ nabla} \ mathrm {A} _ {k} \ right] \ tag {Id-01} \ end {équation} et
\ begin {équation} \ dfrac {\ partial \ left (\ left | \! \ left | \ boldsymbol {\ nabla} \ boldsymbol {\ times} \ mathbf {A} \ right | \! \ right | ^ {2} \ right)} { \ partial \ left (\ boldsymbol {\ nabla} \ mathrm {A} _ {k} \ right)} = \ dfrac {\ partial} {\ partial \ left (\ boldsymbol {\ nabla} \ mathrm {A} _ { k} \ right)} \ left (\ sum ^ {k = 3} _ {k = 1} \ left [\ frac {\ partial \ mathbf {A}} {\ partial x_ {k}} \ boldsymbol {\ cdot } \ boldsymbol {\ nabla} \ mathrm {A} _ {k} - \ Vert \ boldsymbol {\ nabla} \ mathrm {A} _ {k} \ Vert ^ {2} \ right] \ right) = 2 \ left (\ boldsymbol {\ nabla} \ mathrm {A} _ {k} - \ frac {\ partial \ mathbf {A}} {\ partial x_ {k}} \ right) \ tag {Id-02} \ end {équation} où la dérivée fonctionnelle du côté gauche est définie comme \ begin {équation} \ dfrac {\ partial \ left (\ left | \! \ left | \ boldsymbol {\ nabla} \ boldsymbol {\ times} \ mathbf {A} \ right | \! \ right | ^ {2} \ right)} { \ partial \ left (\ boldsymbol {\ nabla} \ mathrm {A} _ {k} \ right)} \ equiv \ left [\ dfrac {\ partial \ left (\ left | \! \ left | \ boldsymbol {\ nabla } \ boldsymbol {\ times} \ mathbf {A} \ right | \! \ right | ^ {2} \ right)} {\ partial \ left (\ dfrac {\ partial \ mathrm {A} _ {k}} { \ partial x_ {1}} \ right)}, \ dfrac {\ partial \ left (\ left | \! \ left | \ boldsymbol {\ nabla} \ boldsymbol {\ times} \ mathbf {A} \ right | \! \ right | ^ {2} \ right)} {\ partial \ left (\ dfrac {\ partial \ mathrm {A} _ {k}} {\ partial x_ {2}} \ right)}, \ dfrac {\ partial \ left (\ left | \! \ left | \ boldsymbol {\ nabla} \ boldsymbol {\ times} \ mathbf {A} \ right | \! \ right | ^ {2} \ right)} {\ partial \ left ( \ dfrac {\ partial \ mathrm {A} _ {k}} {\ partial x_ {3}} \ right)} \ right] \ tag {Id-03} \ end {équation} Preuve de l'équation (Id-01): \ begin {eqnarray *} && \ left | \! \ Left | \ boldsymbol {\ nabla} \ boldsymbol {\ times} \ mathbf {A} \ right | \! \ right | ^ {2} = \ left (\ frac {\ partial A_ {3}} {\ partial x_ {2} } - \ frac {\ partial A_ {2}} {\ partial x_ {3}} \ right) ^ {2} + \ left (\ frac {\ partial A_ {1}} {\ partial x_ {3}} - \ frac {\ partial A_ {3}} {\ partial x_ {1}} \ right) ^ {2} + \ left (\ frac {\ partial A_ {2}} {\ partial x_ {1}} - \ frac {\ partial A_ {1}} {\ partial x_ {2}} \ right) ^ {2} \\ % ---------------------------------------- & = & \ left [\ left (\ frac {\ partial A_ {1}} {\ partial x_ {2}} \ right) ^ {2} + \ left (\ frac {\ partial A_ {1}} {\ partial x_ {3}} \ right) ^ {2} \ right] + \ left [\ left (\ frac {\ partial A_ {2}} {\ partial x_ {1}} \ right) ^ {2} + \ left (\ frac {\ partial A_ {2}} {\ partial x_ {3}} \ right) ^ {2} \ right] + \ left [\ left (\ frac {\ partial A_ {3}} {\ partial x_ {1}} \ right) ^ {2} + \ left (\ frac {\ partial A_ {3}} {\ partial x_ {2}} \ right) ^ {2} \ right] \\ % ---------------------------------------- &&-2 \ left [\ frac {\ partial A_ {1}} {\ partial x_ {2}} \ frac {\ partial A_ {2}} {\ partial x_ {1}} + \ frac {\ partial A_ { 2}} {\ partial x_ {3}} \ frac {\ partial A_ {3}} {\ partial x_ {2}} + \ frac {\ partial A_ {3}} {\ partial x_ {1}} \ frac {\ partial A_ {1}} {\ partial x_ {3}} \ right] \\ % ---------------------------------------- & = & \ left [\ left (\ frac {\ partial A_ {1}} {\ partial x_ {1}} \ right) ^ {2} + \ left (\ frac {\ partial A_ {1}} {\ partial x_ {2}} \ right) ^ {2} + \ left (\ frac {\ partial A_ {1}} {\ partial x_ {3}} \ right) ^ {2} \ right] + \ left [\ left (\ frac {\ partial A_ {2}} {\ partial x_ {1}} \ right) ^ {2} + \ left (\ frac {\ partial A_ {2}} {\ partial x_ {2}} \ droite) ^ {2} + \ gauche (\ frac {\ partial A_ {2}} {\ partial x_ {3}} \ droite) ^ {2} \ droite] \\ % ---------------------------------------- && + \ left [\ left (\ frac {\ partial A_ {3}} {\ partial x_ {1}} \ right) ^ {2} + \ left (\ frac {\ partial A_ {3}} {\ partial x_ {2}} \ right) ^ {2} + \ left (\ frac {\ partial A_ {3}} {\ partial x_ {3}} \ right) ^ {2} \ right] - \ left [\ left ( \ frac {\ partial A_ {1}} {\ partial x_ {1}} \ right) ^ {2} + \ left (\ frac {\ partial A_ {2}} {\ partial x_ {2}} \ right) ^ {2} + \ left (\ frac {\ partial A_ {3}} {\ partial x_ {3}} \ right) ^ {2} \ right] \\ % ---------------------------------------- &&-2 \ left [\ frac {\ partial A_ {1}} {\ partial x_ {2}} \ frac {\ partial A_ {2}} {\ partial x_ {1}} + \ frac {\ partial A_ { 2}} {\ partial x_ {3}} \ frac {\ partial A_ {3}} {\ partial x_ {2}} + \ frac {\ partial A_ {3}} {\ partial x_ {1}} \ frac {\ partial A_ {1}} {\ partial x_ {3}} \ right] \\ % ---------------------------------------- & = & \ Vert \ boldsymbol {\ nabla} \ mathrm {A} _ {1} \ Vert ^ {2} + \ Vert \ boldsymbol {\ nabla} \ mathrm {A} _ {2} \ Vert ^ {2} + \ Vert \ boldsymbol {\ nabla} \ mathrm {A} _ {3} \ Vert ^ {2} - \ left (\ frac {\ partial A_ {1}} {\ partial x_ {1}} \ frac {\ partial A_ {1}} {\ partial x_ {1}} + \ frac {\ partial A_ {2}} {\ partial x_ {1}} \ frac {\ partial A_ {1}} {\ partial x_ {2} } + \ frac {\ partial A_ {3}} {\ partial x_ {1}} \ frac {\ partial A_ {1}} {\ partial x_ {3}} \ right) \\ % ---------------------------------------- &&- \ left (\ frac {\ partial A_ {1}} {\ partial x_ {2}} \ frac {\ partial A_ {2}} {\ partial x_ {1}} + \ frac {\ partial A_ {2 }} {\ partial x_ {2}} \ frac {\ partial A_ {2}} {\ partial x_ {2}} + \ frac {\ partial A_ {3}} {\ partial x_ {2}} \ frac { \ partial A_ {2}} {\ partial x_ {3}} \ right) - \ left (\ frac {\ partial A_ {1}} {\ partial x_ {3}} \ frac {\ partial A_ {3}} {\ partial x_ {1}} + \ frac {\ partial A_ {2}} {\ partial x_ {3}} \ frac {\ partial A_ {3}} {\ partial x_ {2}} + \ frac {\ partial A_ {3}} {\ partial x_ {3}} \ frac {\ partial A_ {3}} {\ partial x_ {3}} \ right) \\ % ---------------------------------------- & = & \ Vert \ boldsymbol {\ nabla} \ mathrm {A} _ {1} \ Vert ^ {2} + \ Vert \ boldsymbol {\ nabla} \ mathrm {A} _ {2} \ Vert ^ {2} + \ Vert \ boldsymbol {\ nabla} \ mathrm {A} _ {3} \ Vert ^ {2} - \ frac {\ partial \ mathbf {A}} {\ partial x_ {1}} \ boldsymbol {\ cdot} \ boldsymbol {\ nabla} \ mathrm {A} _ {1} - \ frac {\ partial \ mathbf {A}} {\ partial x_ {2}} \ boldsymbol {\ cdot} \ boldsymbol {\ nabla} \ mathrm { A} _ {2} - \ frac {\ partial \ mathbf {A}} {\ partial x_ {3}} \ boldsymbol {\ cdot} \ boldsymbol {\ nabla} \ mathrm {A} _ {3} \\ % ---------------------------------------- & = & \ sum ^ {k = 3} _ {k = 1} \ left [\ Vert \ boldsymbol {\ nabla} \ mathrm {A} _ {k} \ Vert ^ {2} - \ frac {\ partial \ mathbf {A}} {\ partial x_ {k}} \ boldsymbol {\ cdot} \ boldsymbol {\ nabla} \ mathrm {A} _ {k} \ right] \ end {eqnarray *} Preuve de l'équation (Id-02): De l'équation \ begin {eqnarray *} && \ left | \! \ Left | \ boldsymbol {\ nabla} \ boldsymbol {\ times} \ mathbf {A} \ right | \! \ right | ^ {2} = \ left (\ frac {\ partial A_ {3}} {\ partial x_ {2} } - \ frac {\ partial A_ {2}} {\ partial x_ {3}} \ right) ^ {2} + \ left (\ frac {\ partial A_ {1}} {\ partial x_ {3}} - \ frac {\ partial A_ {3}} {\ partial x_ {1}} \ right) ^ {2} + \ left (\ frac {\ partial A_ {2}} {\ partial x_ {1}} - \ frac {\ partial A_ {1}} {\ partial x_ {2}} \ right) ^ {2} \\ % ---------------------------------------- & = & \ left [\ left (\ frac {\ partial A_ {1}} {\ partial x_ {2}} \ right) ^ {2} + \ left (\ frac {\ partial A_ {1}} {\ partial x_ {3}} \ right) ^ {2} \ right] + \ left [\ left (\ frac {\ partial A_ {2}} {\ partial x_ {1}} \ right) ^ {2} + \ left (\ frac {\ partial A_ {2}} {\ partial x_ {3}} \ right) ^ {2} \ right] + \ left [\ left (\ frac {\ partial A_ {3}} {\ partial x_ {1}} \ right) ^ {2} + \ left (\ frac {\ partial A_ {3}} {\ partial x_ {2}} \ right) ^ {2} \ right] \\ % ---------------------------------------- &&-2 \ left [\ frac {\ partial A_ {1}} {\ partial x_ {2}} \ frac {\ partial A_ {2}} {\ partial x_ {1}} + \ frac {\ partial A_ { 2}} {\ partial x_ {3}} \ frac {\ partial A_ {3}} {\ partial x_ {2}} + \ frac {\ partial A_ {3}} {\ partial x_ {1}} \ frac {\ partial A_ {1}} {\ partial x_ {3}} \ right] \ end {eqnarray *} nous avons \ begin {eqnarray *} \ dfrac {\ partial \ left (\ left | \! \ left | \ boldsymbol {\ nabla} \ boldsymbol {\ times} \ mathbf {A} \ right | \! \ right | ^ {2} \ right)} { \ partial \ left (\ dfrac {\ partial \ mathrm {A} _ {1}} {\ partial x_ {1}} \ right)} & = & 0 = 2 \ left (\ dfrac {\ partial \ mathrm {A } _ {1}} {\ partial x_ {1}} - \ dfrac {\ partial \ mathrm {A} _ {1}} {\ partial x_ {1}} \ right) \\ \ dfrac {\ partial \ left (\ left | \! \ left | \ boldsymbol {\ nabla} \ boldsymbol {\ times} \ mathbf {A} \ right | \! \ right | ^ {2} \ right)} { \ partial \ left (\ dfrac {\ partial \ mathrm {A} _ {1}} {\ partial x_ {2}} \ right)} & = & 2 \ left (\ dfrac {\ partial \ mathrm {A} _ {1}} {\ partial x_ {2}} - \ dfrac {\ partial \ mathrm {A} _ {2}} {\ partial x_ {1}} \ right) \\ \ dfrac {\ partial \ left (\ left | \! \ left | \ boldsymbol {\ nabla} \ boldsymbol {\ times} \ mathbf {A} \ right | \! \ right | ^ {2} \ right)} { \ partial \ left (\ dfrac {\ partial \ mathrm {A} _ {1}} {\ partial x_ {3}} \ right)} & = & 2 \ left (\ dfrac {\ partial \ mathrm {A} _ {1}} {\ partial x_ {3}} - \ dfrac {\ partial \ mathrm {A} _ {3}} {\ partial x_ {1}} \ right) \ end {eqnarray *} Alors \ begin {équation *} \ dfrac {\ partial \ left (\ left | \! \ left | \ boldsymbol {\ nabla} \ boldsymbol {\ times} \ mathbf {A} \ right | \! \ right | ^ {2} \ right)} {\ partial \ left (\ boldsymbol {\ nabla} \ mathrm {A} _ {1} \ right)} = 2 \ left (\ boldsymbol {\ nabla} \ mathrm {A} _ {1} - \ frac {\ partial\ mathbf {A}} {\ partial x_ {1}} \ right) \ end {équation *} prouvant l'équation (Id-02) pour $ \: k = 1 \: $ et de même pour les deux autres composantes $ \: k = 2,3 $.

Eric Angle
2014-01-03 07:43:22 UTC
view on stackexchange narkive permalink

Une méthode consiste à faire varier l'action de Maxwell (définir $ J ^ \ mu = 0 $ si vous le souhaitez, pour le cas sans source) $$ S = \ int d ^ 4 x {\ mathcal {L}} = - \ int d ^ 4 x \ left (\ frac {1} {4} F ^ {\ mu \ nu} F _ {\ mu \ nu} + J ^ \ mu A_ \ mu \ right). $$ Premièrement, notez que $$ \ begin {align} \ delta \ left (F ^ {\ mu \ nu} F _ {\ mu \ nu} \ right) & = 2 F ^ {\ mu \ nu} \ delta F _ {\ mu \ nu} \\ & = 2 F ^ {\ mu \ nu} \ left (\ partial_ \ mu \ delta A_ \ nu - \ partial_ \ nu \ delta A_ \ mu \ right) \\ & = 4 F ^ {\ mu \ nu } \ partial_ \ mu \ delta A_ \ nu \\ & = 4 \ left [\ partial_ \ mu \ left (F ^ {\ mu \ nu} \ delta A_ \ nu \ right) - \ partial_ \ mu F ^ {\ mu \ nu} \ delta A_ \ nu \ right], \ end {align} $$ où nous avons utilisé le fait que $ F $ est antisymétrique.

Notez également que le $ \ partial_ \ mu \ left (F ^ {\ mu \ nu} \ delta A_ \ nu \ right) $ term disparaîtra lors de sa conversion en intégrale de surface, en utilisant l'argument standard que $ \ delta A_ \ mu $ disparaît à la limite d'intégration.

En utilisant ce qui précède, la variation de l'action est $$ \ delta S = - \ int d ^ 4x \ \ delta A_ \ nu \ left (- \ p artial_ \ mu F ^ {\ mu \ nu} + J ^ \ nu \ right), $$ qui, puisque $ \ delta A_ \ nu $ est arbitraire, donne le résultat souhaité $$ \ partial_ \ mu F ^ {\ mu \ nu} = J ^ \ nu. $$

Au lieu de supposer que $ \ delta A_ \ mu $ disparaît à la frontière, on peut supposer $ F ^ {\ mu \ nu} $ à la frontière.Est-ce faux?Voir une question connexe ici https://physics.stackexchange.com/questions/438277/is-it-enough-to-assume-f-mu-nu-to-0-at-infinity-but-not-a-mu-pour-dériver-t @EricAngle


Ce Q&R a été automatiquement traduit de la langue anglaise.Le contenu original est disponible sur stackexchange, que nous remercions pour la licence cc by-sa 2.0 sous laquelle il est distribué.
Loading...